What is the equation of a line that passes through the point (1,8) and is perpendicular to the line whose equation is y=x/2+3?

Answers

Answer 1

Answer:

m=1/2

y-8=1/2(x-1)

y-8=1/2x-1/2

multiply through by 2

2y-16=x-1

2y-16+1-x=0

2y-15-x=0

2y-x-15=0


Related Questions

Find the area of ​​a 10 cm sphere
.
help

Answers

Answer:

that's 4,188.8 if it's gonna be a 10cm sphere

can you please answer this????

Answers

Answer:

x = 5

Step-by-step explanation:

I'm taking all bases as b so not typing it

2/3 log 125 = log (125^2/3) = log 25

1/2 log 9 = log (9^1/2) = log 3

So we can rewrite the equation as,

log x = log 25 + log 3 - log 15

or, log x = log (25×3) - log 15

or, log x = log 75 - log 15

or, log x = log (75/15)

or, log x = log 5

or, x = 5

Answered by GAUTHMATH

Helen is constructing a room. She is preparing a scale drawing of her room as 1 cm = 2.5 feet. Find the actual dimensions with the given model dimensions of 8 cm×5 cm.

20 feet×12.5 feet
15 feet×5.5 feet
10 feet×8 feet
8 feet×6.5 feet

Answers

Answer: 20 ft × 12.5 ft

Step-by-step explanation:

Since 1 cm = 2.5 ft,

8 cm = 8 · 2.5 = 20 ft5 cm = 5 · 2.5 = 12.5 ft

Therefore, 8 cm × 5 cm = 20 ft × 12.5 ft

Identify the first 4 terms in the arithmetic sequence given by the explicit formula ƒ(n) = 8 + 3(n – 1).

Answers

Answer:

Step-by-step explanation:

f(n) = 8 + 3(n) - 3

f(n) = 5 + 3n

f(1) = 5 + 3(1)

f(1) = 8

f(2) = 5 + 3(2)

f(2) = 5  + 6

f(2) = 11

f(3) = 5 + 3*3

f(3) = 14

f(4) = 5 + 3*4

f(4) = 17

5. Sam wrote the expression below.
10 +15k
Rami said that this expression is equivalent to 5(3k + a)
Kenneth said this expression is equivalent toyk+6+8k+4.
Who is correct and why? Explain your thinking clearly,

Answers

Answer:

see below

Step-by-step explanation:

10 + 15k

Factor out the greatest common factor 5

5( 2+3k)

Rewriting

5(3k+2)

Rami is correct if a=2 then his expression is 5(3k+2)

Kenneth

yk+6+8k+4

Add the terms together

k(y+8) + 10

If y =7 then Kenneth is correct otherwise he is incorrect

Sam's monthly bills are normally distributed with mean 2700 and standard deviation 230.9. He receives two paychecks of $1500 each in a month, post taxes and withholdings. What is the probability that his expenses will exceed his income in the following month?Ð) 10%. B) 16%.C) 21%.D) 29%.E) 37%.

Answers

Answer:

A) 10%

Step-by-step explanation:

Normal Probability Distribution

Problems of normal distributions can be solved using the z-score formula.

In a set with mean [tex]\mu[/tex] and standard deviation [tex]\sigma[/tex], the z-score of a measure X is given by:

[tex]Z = \frac{X - \mu}{\sigma}[/tex]

The Z-score measures how many standard deviations the measure is from the mean. After finding the Z-score, we look at the z-score table and find the p-value associated with this z-score. This p-value is the probability that the value of the measure is smaller than X, that is, the percentile of X. Subtracting 1 by the p-value, we get the probability that the value of the measure is greater than X.

Sam's monthly bills are normally distributed with mean 2700 and standard deviation 230.9.

This means that [tex]\mu = 2700, \sigma = 230.9[/tex]

What is the probability that his expenses will exceed his income in the following month?

Expenses above 2*1500 = $3000, which is 1 subtracted by the p-value of Z when X = 3000.

[tex]Z = \frac{X - \mu}{\sigma}[/tex]

[tex]Z = \frac{3000 - 2700}{230.9}[/tex]

[tex]Z = 1.3[/tex]

[tex]Z = 1.3[/tex] has a p-value of 0.9032.

1 - 0.9032 = 0.0968 that is, close to 10%, and thus the correct answer is given by option A.

Suppose that the value of a stock varies each day from $12.82 to $28.17 with a uniform distribution.
Find the third quartile; 75% of all days the stock is below what value? (Enter your answer to the nearest cent.)

Answers

Answer:  24.33

======================================================

Explanation:

The range is

range = max - min

range = 28.17 - 12.82

range = 15.35

This is the width of this particular uniform distribution.

Apply 75% to this value

75% of 15.35 = 0.75*15.35 = 11.5125

Then finally, add that to the min

12.82 + 11.5125 = 24.3325 which rounds to 24.33

We can see that 75% of the values are below 24.33 which makes it the 3rd quartile (Q3).

A random sample of 25 values is drawn from a mound-shaped and symmetric distribution. The sample mean is 9 and the sample standard deviation is 2. Use a level of significance of 0.05 to conduct a two-tailed test of the claim that the population mean is 8.5.(a) Is it appropriate to use a Student's t distribution? Explain.Yes, because the x distribution is mound-shaped and symmetric and Ï is unknown.No, the x distribution is skewed left. No, the x distribution is skewed right.No, the x distribution is not symmetric.No, Ï is known.How many degrees of freedom do we use?(b) What are the hypotheses?H0: μ = 8.5; H1: μ > 8.5H0: μ = 8.5; H1: μ â  8.5 H0: μ = 8.5; H1: μ < 8.5H0: μ < 8.5; H1: μ = 8.5H0: μ > 8.5; H1: μ = 8.5(c) Compute the t value of the sample test statistic. (Round your answer to three decimal places.)t =(d) Estimate the P-value for the test.P-value > 0.2500.100 < P-value < 0.250 0.050 < P-value < 0.1000.010 < P-value < 0.050P-value < 0.010(e) Do we reject or fail to reject H0?At the α = 0.05 level, we reject the null hypothesis and conclude the data are statistically significant.At the α = 0.05 level, we reject the null hypothesis and conclude the data are not statistically significant. At the α = 0.05 level, we fail to reject the null hypothesis and conclude the data are statistically significant.At the α = 0.05 level, we fail to reject the null hypothesis and conclude the data are not statistically significant.(f) Interpret the results.There is sufficient evidence at the 0.05 level to reject the null hypothesis.There is insufficient evidence at the 0.05 level to reject the null hypothesis.

Answers

Answer:

1.) Yes, because the x distribution is mound-shaped and symmetric and σ is unknown. ;

df = 24 ;

H0 : μ = 8.5

H1 : μ ≠ 8.5 ;

1.250 ;

At the α = 0.05 level, we fail to reject the null hypothesis and conclude the data are not statistically significant.

There is insufficient evidence at the 0.05 level to reject the null hypothesis.

Step-by-step explanation:

Given :

Sample size, n = 25

xbar = 9 ; Standard deviation, s = 2

α = 0.05 ;

The degree of freedom, df = n - 1 ; 25 - 1 = 24

The hypothesis (two tailed)

H0 : μ = 8.5

H1 : μ ≠ 8.5

The test statistic :

(xbar - μ) ÷ (s/√(n))

(9 - 8.5) ÷ (2/√(25))

0.5 / 0.4

Test statistic = 1.250

The Pvalue from Tscore ;

Pvalue(1.250, 24) = 0.2234

Pvalue > α ; We fail to reject H0 ;

Suppose that the functions and g are defined for all real numbers x as follows.
f(x)=x+6
g(x) = 2x + 6
Write the expressions for (f-g)(x) and (fg)(x) and evaluate (f+g)(1).

Answers

Answer:

Step-by-step explanation:

Given functions are,

f(x) = x + 6

g(x) = 2x + 6

(f - g)(x) = (x + 6) - (2x + 6)

                   = -x

(f . g)(x) = f(x) × g(x)

            = (x + 6)(2x + 6)

            = 2x² + 6x + 12x + 36

            = 2x² + 18x + 36

(f + g)(x) = (x + 6) + (2x + 6)

             = 3x + 12

(f + g)(1) = 3(1) + 12

            = 15  

Find the equation of the line passing through (4,1) and perpendicular to the line whose equation is 1x-3y-4=0

Answers

9514 1404 393

Answer:

  3x +y -13 = 0

Step-by-step explanation:

The perpendicular line will have the variable coefficients swapped and one of them negated. The new constant will be appropriate to the given point.

  3(x -4) +1(y -1) = -0

  3x +y -13 = 0

_____

Additional comment

The given equation is in "general form", so that is the form of the equation we have given as the answer. This form is convenient in that the general form equation for a line through the origin, ax+by=0, is easily translated to make it pass through a point (h, k): a(x -h) +b(y -k) = 0. Eliminating parentheses puts the equation back into general form.

What is the slope of the line?
-3

-1/3

1/3

3

Answers

Answer:

D) 3

Step-by-step explanation:

Rise/run, rise is 3, run is 1

Answer:

3

Step-by-step explanation:

Pick two points on the line

(0,0) and ( 1,3)

The slope is found by

m = ( y2-y1)/(x2-x1)

    = ( 3-0)/(1-0)

    = 3/1

   = 3

Y
X

Pls help me you’ll get 29 points

Answers

Answer:

x = 60

Step-by-step explanation:

The sum of the angles of a triangle add to 180

x+x+x = 180

3x = 180

Divide by 3

3x/3 =180/3

x = 60

What is the value of the expression 10(n-6) when 4=14

Answers

Answer:

it's going to be 4n so its going to be 4...

Step-by-step explanation:

Answer:

80

Step-by-step explanation:

Assuming you meant when n=14

10(n-6)

plug in 14 for n

10(14-6)

work out parenthesis first

10(8)=

80

Use the​ Gauss-Jordan method to solve the system of equations. If the system has infinitely many​ solutions, give the solution with z arbitrary

Answers

Write each equation in standard form:

3x + y + 3z = 11

x + 2y + z = 7

-x + y + z = 0

In matrix form, this is

[tex]\begin{bmatrix}3&1&3\\1&2&1\\-1&1&1\end{bmatrix}\begin{bmatrix}x\\y\\z\end{bmatrix}=\begin{bmatrix}11\\7\\0\end{bmatrix}[/tex]

and in augmented matrix form,

[tex]\left[\begin{array}{ccc|c}3&1&3&11\\1&2&1&7\\-1&1&1&0\end{bmatrix}\right][/tex]

Now for the row operations:

• Add row 1 to -3 (row 2), and add row 1 to 3 (row 3):

[tex]\left[\begin{array}{ccc|c}3&1&3&11\\0&-5&0&-10\\0&4&6&11\end{bmatrix}\right][/tex]

• Multiply row 2 by -1/5:

[tex]\left[\begin{array}{ccc|c}3&1&3&11\\0&1&0&2\\0&4&6&11\end{bmatrix}\right][/tex]

• Add -4 (row 2) to row 3:

[tex]\left[\begin{array}{ccc|c}3&1&3&11\\0&1&0&2\\0&0&6&3\end{bmatrix}\right][/tex]

• Multiply row 3 by 1/6:

[tex]\left[\begin{array}{ccc|c}3&1&3&11\\0&1&0&2\\0&0&1&\frac12\end{bmatrix}\right][/tex]

• Add -1 (row 2) and -3 (row 3) to row 1:

[tex]\left[\begin{array}{ccc|c}3&0&0&\frac{15}2\\0&1&0&2\\0&0&1&\frac12\end{bmatrix}\right][/tex]

• Mutiply row 1 by 1/3:

[tex]\left[\begin{array}{ccc|c}1&0&0&\frac52\\0&1&0&2\\0&0&1&\frac12\end{bmatrix}\right][/tex]

Then the solution to the system is (x, y, z) = (5/2, 2, 1/2).

Find the product of these complex numbers.
(8 + 5)(6 + 3) =

Answers

The Answer is D, 33+54i!
Answer:

D

Step-by-step explanation:

For this kind of stuff, we’ll need to use the FOIL Method. Basically start with the Firsts, Outers, Inners and Lasts.

(8 + 5i) (6 + 3i) =

Multiply 8 x 6

Answer: 48

Multiply 5i x 3i

Answer: 15i^2

Multiply 5i x 6

Answer: 30i

Multiply 8 x 3i

Answer: 24i

Then add 30i + 24i = 54i
Finally subtract 48 - 15 = 33

Final Answer: 33 + 54i

821) The integon which is 15 more than - 55 is

Answers

Answer:

-40

Step-by-step explanation:

-55 + 15 = x

-40 =x

Solve the inequality (help please)

Answers

Answer:

v<1 23/25

Step-by-step explanation:

The inequality simplifies to 48/25, which is equivalent to 1 23/25.

I need help:/ I’m in college

Answers

Step-by-step explanation:

Amount of acid = 14.9% of 331 mL solution

= 0.149×(331 mL)

= 49.3 mL acid

Blood pressure values are often reported to the nearest 5 mmhg (100, 105, 110, etc.). the actual blood pressure values for nine randomly selected individuals are given below.

108.6 117.4 128.4 120.0 103.7 112.0 98.3 121.5 123.2

Required:
a. What is the median of the reported blood pressure values?
b. Suppose the blood pressure of the second individual is 117.7 rather than 117.4 (a small change in a single value). What is the new median of the reported values?
c. What does this say about the sensitivity of the median to rounding or grouping in the data?

Answers

Answer:

Step-by-step explanation:

Arranging the data in the ascending order:

108.6 98.3 103.7 112 117.4 120 121.5 123.2 128.4

The median is the middle value of the data set:

a)

Hence,

median = 117.4

b)

When the value of blood pressure is 117.7 instead of 117.4 then the median will be:

Median = 117.7

c)

This indicates that the median of a well sorted set of data is depends upon the middle value of the data set.

Answer question below

Answers

Answer:

edge of cube = root 13.5/6 = 1.5

volume of cube = (Edge of cube)^3= (1.5)^3 = 3.37500 m^3

PLEASE PLEASE HELP ASAPPPP IM BEING TIMEDDD

6x2y − 3xy − 24xy2 + 12y2
Rewrite the expression completely factored. Show the steps of your work.

Answers

Answer:

3y(2x-1)(x-4y)

Step-by-step explanation:

Apply exponent rule:

6x^2y-3xy-24xyy+12yy

Rewrite 12 as 4*3

Rewrite -24 as 8*3

Rewrite 6 as 2*3

2*3x^2y-3xy+8*3xyy+4*3yy

Factor out common term 3y:

3y(2x^2-x-8xy+4y)

Factor 2x^2-x-8xy+4y:

3y(2x-1)(x-4y)

Your Answer Is 3y(2x-1)(x-4y)

4. Five cards are randomly chosen from a deck of 52 (13 denominations with 4 suits). a. How many ways are there to receive 5 cards from a deck of 52

Answers

Answer:

There are 2,598,960 ways to receive 5 cards from a deck of 52.

Step-by-step explanation:

The order in which the cards are chosen is not important, which means that the combinations formula is used to solve this question.

Combinations formula:

[tex]C_{n,x}[/tex] is the number of different combinations of x objects from a set of n elements, given by the following formula.

[tex]C_{n,x} = \frac{n!}{x!(n-x)!}[/tex]

a. How many ways are there to receive 5 cards from a deck of 52?

[tex]C_{52,5} = \frac{52!}{5!(47)!} = 2598960[/tex]

There are 2,598,960 ways to receive 5 cards from a deck of 52.

The diameter of the base is the cone measured 8 units. The height measures 6 units.

What is the volume of the cone?

A) 24 π cubic units

B) 32 π cubic units

C)48 π cubic units

D)64 π cubic units

Answers

The answer is B) 32 cubic units

What is the range for the following set of numbers?57, -5, 11, 39, 56, 82, -2, 11, 64, 18, 37, 15, 68

Answers

The range= the highest number - the lowest number

so

82-(-2)

=84

then ur answer is 84

If 400 patrons visit the park in March and 550 patrons visit in April, the total number of patrons who
visited the park over the two months falls into all of the following categories except
O real numbers
O rational numbers
o irrational numbers

Answers

950 is a real number, and it is a rational number since it can be expressed as 950/1. Therefore, it falls into all of the categories expect irrational numbers.

Two planes are the same altitude. From the airport , one plane is 50 km away in the direction of N°60 E and another is 80 km away in the direction of S50° E .How far apart are the two planes

Answers

9514 1404 393

Answer:

  78.5 km

Step-by-step explanation:

Measured at the airport, the angle between the two planes is ...

  180° -60° -50° = 70°

The law of cosines tells us the distance between the planes is ...

  d = √(50² +80² -2·50·80·cos(70°)) ≈ √6163.84 ≈ 78.5 . . . km

The planes are about 78.5 km apart.

Matt buys a new fish tank. The fish tank is in the shape of a cuboid. The diagram shows water in the tank. 30 cm 30 cm 100 cm Matt knows 1000 cm' = 1 litre 1 gallons = 4.5 litres He can keep 2 small fish in the tank for every 1 gallon of water in the tank. Matt thinks he can keep more than 36 small fish in the tank. Is Matt correct?​

Answers

Answer: Yes, but only if he houses 37, 38, 39, or 40 fish

Anything larger than 40 and he'll need more room.

==========================================================

Explanation:

The tank is 30 cm by 30 cm by 100 cm. The volume is 30*30*100 = 90,000 cm^3 which is shorthand for "cubic centimeters".

We're told that 1000 cm^3 = 1 liter, which means the 90,000 cm^3 converts to (90,000)/(1000) = 90 liters.

The fish tank is 90 liters.

Since 1 gallon = 4.5 liters, this means the 90 liter tank converts to 90/(4.5) = 20 gallons

----------------------------

Your teacher mentions "He can keep 2 small fish for every 1 gallon".

Since the tank is 20 gallons, that means he can keep 20*2 = 40 fish. This value is larger than 36, so Matt is correct to a point. If Matt is thinking 37, 38, 39, or 40 fish then he would be correct. If Matt is wanting more than 40 fish, then he'll need a bigger tank.

In short, he can't have any number over 36 and can only have 4 specific values (the four values mentioned earlier).

So technically, Matt is correct, but strong clarification is needed.

Hello everyone can someone answer this question please

Answers

9514 1404 393

Answer:

  (a)  2

Step-by-step explanation:

Each inch is 2.54 cm, so 5.08 cm is ...

  x / (5.08 cm) = (1 in) / (2.54 cm)

  x = (1 in)(5.08/2.54) = (1 in)(2)

  x = 2 in

5.08 cm equals 2 inches.

Write the equation of each line in slope intercept form. Slope is -6, and (1,-2) is on the line

Answers

Y = -6x + 4

See the attached photo for further reference.

Hope this helps! Please make me the brainliest, it’s not necessary but appreciated, I put a lot of effort and research into my answers. Have a good day, stay safe and stay healthy.

Find the lengths the missing side

Answers

Answer:

Long leg = 10√3Short leg = 10Hypotenuse = 20

Step-by-step explanation:

Concept:

Here, we need to know the idea of a special triangle 30-60-90.

A special right triangle is a right triangle with some regular feature that makes calculations on the triangle easier, or for which simple formulas exist.

The ratio between the corresponding side of each angle is 1 : √3 : 2

If you are still confused, please refer to the attachment below for a graphical explanation.

Solve:

30-60-90 ⇔ 1 : √3 : 2

Given the side corresponding to 90° is 20

30° : 90° = 1 : 2

30° : 20 = 1 : 2

30° = 10

30° : 60° = 1 : √3

10 : 60° = 1 : √3

60° = 10√3

Hope this helps!! :)

Please let me know if you have any questions

Other Questions
Write the cell notation for an electrochemical cell consisting of an anode where Mn (s) is oxidized to Mn2 (aq) and a cathode where Co2 (aq) is reduced to Co (s) . Assume all aqueous solutions have a concentration of 1 mol/L. which group settled mostly on crete? PLEASE PLEASE HELP ASAPPPP IM BEING TIMEDDD6x2y 3xy 24xy2 + 12y2Rewrite the expression completely factored. Show the steps of your work. Need a well written paragraph. Origin- How did the human rights issue come about? Who does it affect and how? How does it affect human rights? Which of the following best describes bacteriophage?wA.A bacterium that obtains energy by feeding on other bacteriaB.A virus that infects bacteriaC.A virus that passes eukaryotic genes to prokaryotesD.A bacterium that takes in free DNA from its environment What is the length of the hypotenuse in the right triangle shown below?45456 Name the following bases.Fe(OH)3- iron(III) hydroxide- iron(II) hydroxide- iron hydroxide Which of the following is an important consideration in setting up the proper environment to complete tasks on time? How are all fungi and all protists alike? 4. Five cards are randomly chosen from a deck of 52 (13 denominations with 4 suits). a. How many ways are there to receive 5 cards from a deck of 52 The Marbury v. Madison ruling changed the U.S. government by:A. allowing states to pass laws that violated federal laws.B. establishing the president's power to veto laws passed byCongress.C. preventing Congress from impeaching a siting president.D. giving the Supreme Court the right to rule laws unconstitutional. A 4.0kg brick is sliding on a surface. The coefficient of kinetic friction between the surfaces is 0.25. What is the size of the force of friction?a. ON b. 1 Nc. 10 Nd. 4 N On a tuesday a magician said i made my wife disappear 31 days ago what day of the week did he make her disappear Which best describes the diction in "Ain't I a Woman?"o formal and detachedO formal and scholarlyinformal and relatableO informal and light-hearted How do human activities lead to change in the weather? 2/3 of a number is 11 work out half the number HELPPPP, which expression is it? ive came so far and im almost there.. please help!! What is the solution to the equation?3 + V3x 5= x simplify the following[tex]simplify \: the \: follwing \: \\ logx \: x9[/tex]please I need help